11
$\begingroup$

There is a simple question where a unit square contains four circles of radius $\frac14$ with a fifth inner circle, and the aim is to find that the radius of the inner circle is $r_2=\frac{\sqrt{2}-1}{4} \approx 0.10355$. Consider the line segment from a vertex passing through the centre of an outer circle to the centre of the square.

circles in box

Going up a dimension, with a unit cube with eight spheres of radius $\frac14$ and a ninth inner sphere, it is not much harder to find the radius of the inner sphere is $r_3=\frac{\sqrt{3}-1}{4} \approx 0.18301$. And the simple geometry looking at the distance from a vertex of the cube to its centre generalises further: with a unit hypercube with $2^n$ hyperspheres of radius $\frac14$ and an inner hypersphere, the radius of the inner hypersphere is $r_n=\frac{\sqrt{n}-1}{4}$.

For $n=4$ this gives $r_4=\frac14$ so the inner hypersphere is then the same size as the sixteen outer hyperspheres. For $n=9$ it gives $r_9=\frac12$ so the inner hypersphere touches the hypercube.

Counterintuitively, for $n=10$ this gives $r_{10}\approx 0.54057$ so the inner hypersphere goes outside the hypercube which bounds the outer hyperspheres which bound the inner hypersphere, and similarly for larger $n$.

The $n$-volume of the hypercube is $1$. The $n$-volume of the inner $n$-ball is $\frac{\left(\sqrt{\pi}\frac{\sqrt{n}-1}{4}\right)^n}{\Gamma\left(\frac n2+1\right)}$: it is $0$ for $n=1$, about $0.033688$ for $n=2$, then decreasing as $n$ increases to $264$ where it is about $10^{-5}$, and then increases with $n$, exceeding $1$ when $n\ge 1206$ and exceeding $2$ when $n \ge 1244$.

For what dimension $n$ is more than half of the inner $n$-ball outside the hypercube?

Calculating the volumes of the $2n$ hyperspherical caps may not lead to the answer, as it seems that for large enough $n$ (possibly $n\ge 15\gt 9+\sqrt{32} \approx 14.657$ and certainly $n\ge 17$) the hyperspherical caps may overlap each other.

Simulation using the R code below suggests $n\ge 43$ will work, but this is not an analytic answer.

pointinnball <- function(n, r=1){
  x <- rnorm(n)
  rmsx <- sqrt(sum(x^2))
  x * r * runif(1)^(1/n) / rmsx
  }
innerradius <- function(n){
  (sqrt(n) - 1) / 4
  }
pointoutsidecube <- function(n){
  max(abs(pointinnball(n, r=innerradius(n)))) > 1/2
  }

set.seed(2025)
mean(replicate(10^6, pointoutsidecube(n=42)))
# 0.494222
mean(replicate(10^6, pointoutsidecube(n=43)))  
# 0.512103
$\endgroup$
5
  • 1
    $\begingroup$ It must be a nightmare to compute a close form of the volume! Nice problem though $\endgroup$ Commented Aug 28 at 15:11
  • 2
    $\begingroup$ related intersection of hypercube and hypersphere $\endgroup$ Commented Aug 30 at 16:12
  • $\begingroup$ @user953715 following that link and links from it, I came across an expression in (2.6) in arxiv.org/pdf/1804.07861 (Peter J. Forrester, "Comment on 'Sum of squares of uniform random variables' by I. Weissman") for the intersection of a hypercube and $n$-ball but my attempts to use it to answer this question failed miserably. $\endgroup$ Commented Sep 4 at 13:25
  • $\begingroup$ @Henry Unfortunately, it seems the user had deleted their account recently. $\endgroup$ Commented Sep 4 at 13:27
  • 1
    $\begingroup$ @Soheil It was still a helpful comment including the diagram in the linked question of overlapping spherical caps, and especially the links from LeoDucas and E. Postlethwaite, even if I failed when trying to implement some of the referenced expressions. $\endgroup$ Commented Sep 5 at 22:32

1 Answer 1

2
+25
$\begingroup$

Here's some progress: I can show that this holds for $n\geq 98$.


We will scale everything up by $2$. Let $V_n$ be the volume of an $n$-ball of radius $(\sqrt n-1)/2$, and let $W_n$ be the volume of the intersection of said $n$-ball with the hypercube $[-1,1]^n$. We are concerned with when $W_n/V_n<1/2$.

Note that $W_n/2^n$ is the probability that, if $x_1,\ldots,x_n\sim\operatorname{Unif}([-1,1])$ are chosen independently, then $x_1^2+\cdots+x_n^2\leq (\sqrt n-1)^2/4=(n-2\sqrt n+1)/4$. Let $D$ be the distribution of each $x_i^2$, and let $y_i=x_i^2$ so that $y_1,\ldots,y_n\sim D$. Note that $\mathbb E[y_i]=1/3$. Write $$\delta=\delta_n=1-\frac{\left(\frac{\sqrt n-1}2\right)^2}{\frac n3},$$ so that $$2^{-n}W_n=\Pr\big[y_1+\cdots+y_n<(1-\delta)\mathbb E(y_1+\cdots+y_n)\big].$$ We now run a Chernoff-bound calculation manually. Let $\lambda=\lambda_n>0$ be a parameter to choose later. We have \begin{align*} \Pr\big[y_1+\cdots+y_n<(1-\delta)\mathbb E(y_1+\cdots+y_n)\big] &\leq\frac{\mathbb E(e^{-\lambda(y_1+\cdots+y_n)})}{e^{-\lambda(1-\delta)n/3}}\\ &=e^{\lambda(1-\delta)n/3}\left(\mathbb E_{y\sim D}(e^{-\lambda y})\right)^n\\ &=\left(e^{\lambda(1-\delta)/3}\int_0^1e^{-\lambda x^2}dx\right)^n. \end{align*} Therefore, for any $\lambda>0$, $$\frac{W_n}{V_n}\leq\left(\frac n2\right)!\cdot\left(\frac{2}{\sqrt\pi\cdot\frac{\sqrt n-1}2}e^{\lambda(1-\delta)/3}\int_0^1e^{-\lambda x^2}dx\right)^n.$$ This is strong enough to prove that $W_n/V_n<1/2$ when $n=98$ by taking $\lambda=1.83$. This computation is laid out in this Desmos snapshot.

$\endgroup$
1
  • 1
    $\begingroup$ The same R simulation suggests that with $n=98$ about $95.6\%$ of the volume of the inner $n$-ball may be outside the hypercube. $\endgroup$ Commented Sep 3 at 7:39

You must log in to answer this question.

Start asking to get answers

Find the answer to your question by asking.

Ask question

Explore related questions

See similar questions with these tags.